Difference between revisions of "2007 AIME II Problems/Problem 6"

m (What's with the "noinclude" tags in this question?)
m (Solution)
Line 9: Line 9:
 
{| align=center style="border:1px solid black;"
 
{| align=center style="border:1px solid black;"
 
|-  
 
|-  
| Number   || 1st   || 2nd   || 3rd   || 4th  
+
| Digit   || 1st   || 2nd   || 3rd   || 4th  
 
|-
 
|-
 
| 0 || 0 || 0 || 0 || 64
 
| 0 || 0 || 0 || 0 || 64
Line 32: Line 32:
 
|}
 
|}
  
For any number from 1-8, there are exactly 4 numbers from 1-8 that are odd and less than the number or that are even and greater than the number (the same will happen for 0 and 9 in the last column). Thus, the answer is <math>\displaystyle 4^{k-1} \cdot 10 = 4^3\cdot10 = 640</math>.
+
For any number from 1-8, there are exactly 4 numbers from 1-8 that are odd and less than the number or that are even and greater than the number (the same will happen for 0 and 9 in the last column). Thus, the answer is <math>4^{k-1} \cdot 10 = 4^3\cdot10 = 640</math>.
  
 
== See also ==
 
== See also ==

Revision as of 14:33, 21 December 2011

Problem

An integer is called parity-monotonic if its decimal representation $a_{1}a_{2}a_{3}\cdots a_{k}$ satisfies $a_{i}<a_{i+1}$ if $a_{i}$ is odd, and $a_{i}>a_{i+1}$ if $a_{i}$ is even. How many four-digit parity-monotonic integers are there?

Solution

Let's set up a table of values. Notice that 0 and 9 both cannot appear as any of $a_1,\ a_2,\ a_3$ because of the given conditions. A clear pattern emerges.

For example, for $3$ in the second column, we note that $3$ is less than $4,6,8$, but greater than $1$, so there are four possible places to align $3$ as the second digit.

Digit   1st   2nd   3rd   4th
0 0 0 0 64
1 1 4 16 64
2 1 4 16 64
3 1 4 16 64
4 1 4 16 64
5 1 4 16 64
6 1 4 16 64
7 1 4 16 64
8 1 4 16 64
9 0 0 0 64

For any number from 1-8, there are exactly 4 numbers from 1-8 that are odd and less than the number or that are even and greater than the number (the same will happen for 0 and 9 in the last column). Thus, the answer is $4^{k-1} \cdot 10 = 4^3\cdot10 = 640$.

See also

2007 AIME II (ProblemsAnswer KeyResources)
Preceded by
Problem 5
Followed by
Problem 7
1 2 3 4 5 6 7 8 9 10 11 12 13 14 15
All AIME Problems and Solutions